Clarification of assumptions made in deriving error of implicit midpoint rule












1














In my derivation for $y^prime = f(t,y)$, I begin by writing the method as an expression which should simplify to the error, by substitution of the exact solution
begin{equation} y(t_{n+1}) - y(t_n) - hfleft(t_n+frac{1}{2}h,,frac{1}{2}(y(t_{n+1}) + y(t_n))right)end{equation}
From here, it may be simplified in a straightforward way using Taylor expansions to the following
begin{equation}hy^prime(t_n) + frac{1}{2}h^2y^{primeprime}(t_n) + mathcal{O}(h^3) - hfleft(t_n+frac{1}{2}h,,yleft(t_{n}+frac{1}{2}hright)+ mathcal{O}(h^2)right)end{equation}
This is where I am unsure how to proceed. From this question, the top answer seems to suggest that it is a trivial matter to assume that we may assert $$f(t_n + 0.5h,y(t_n + 0.5h) + mathcal{O}(h^2)) = f(t_n + 0.5h,y(t_n + 0.5h)) + mathcal{O}(h^2) = y^prime(t_n+0.5h) + mathcal{O}(h^2)$$
but this does not seem obvious to me. While it is stipulated that $f$ is an analytic function, I don't see how a perturbation in the input would necessarily perturb the output on the order of $h^2$ in general.



If this is a good assumption to make, could someone please explain? If not, why? What is the correct way to proceed here?










share|cite|improve this question





























    1














    In my derivation for $y^prime = f(t,y)$, I begin by writing the method as an expression which should simplify to the error, by substitution of the exact solution
    begin{equation} y(t_{n+1}) - y(t_n) - hfleft(t_n+frac{1}{2}h,,frac{1}{2}(y(t_{n+1}) + y(t_n))right)end{equation}
    From here, it may be simplified in a straightforward way using Taylor expansions to the following
    begin{equation}hy^prime(t_n) + frac{1}{2}h^2y^{primeprime}(t_n) + mathcal{O}(h^3) - hfleft(t_n+frac{1}{2}h,,yleft(t_{n}+frac{1}{2}hright)+ mathcal{O}(h^2)right)end{equation}
    This is where I am unsure how to proceed. From this question, the top answer seems to suggest that it is a trivial matter to assume that we may assert $$f(t_n + 0.5h,y(t_n + 0.5h) + mathcal{O}(h^2)) = f(t_n + 0.5h,y(t_n + 0.5h)) + mathcal{O}(h^2) = y^prime(t_n+0.5h) + mathcal{O}(h^2)$$
    but this does not seem obvious to me. While it is stipulated that $f$ is an analytic function, I don't see how a perturbation in the input would necessarily perturb the output on the order of $h^2$ in general.



    If this is a good assumption to make, could someone please explain? If not, why? What is the correct way to proceed here?










    share|cite|improve this question



























      1












      1








      1







      In my derivation for $y^prime = f(t,y)$, I begin by writing the method as an expression which should simplify to the error, by substitution of the exact solution
      begin{equation} y(t_{n+1}) - y(t_n) - hfleft(t_n+frac{1}{2}h,,frac{1}{2}(y(t_{n+1}) + y(t_n))right)end{equation}
      From here, it may be simplified in a straightforward way using Taylor expansions to the following
      begin{equation}hy^prime(t_n) + frac{1}{2}h^2y^{primeprime}(t_n) + mathcal{O}(h^3) - hfleft(t_n+frac{1}{2}h,,yleft(t_{n}+frac{1}{2}hright)+ mathcal{O}(h^2)right)end{equation}
      This is where I am unsure how to proceed. From this question, the top answer seems to suggest that it is a trivial matter to assume that we may assert $$f(t_n + 0.5h,y(t_n + 0.5h) + mathcal{O}(h^2)) = f(t_n + 0.5h,y(t_n + 0.5h)) + mathcal{O}(h^2) = y^prime(t_n+0.5h) + mathcal{O}(h^2)$$
      but this does not seem obvious to me. While it is stipulated that $f$ is an analytic function, I don't see how a perturbation in the input would necessarily perturb the output on the order of $h^2$ in general.



      If this is a good assumption to make, could someone please explain? If not, why? What is the correct way to proceed here?










      share|cite|improve this question















      In my derivation for $y^prime = f(t,y)$, I begin by writing the method as an expression which should simplify to the error, by substitution of the exact solution
      begin{equation} y(t_{n+1}) - y(t_n) - hfleft(t_n+frac{1}{2}h,,frac{1}{2}(y(t_{n+1}) + y(t_n))right)end{equation}
      From here, it may be simplified in a straightforward way using Taylor expansions to the following
      begin{equation}hy^prime(t_n) + frac{1}{2}h^2y^{primeprime}(t_n) + mathcal{O}(h^3) - hfleft(t_n+frac{1}{2}h,,yleft(t_{n}+frac{1}{2}hright)+ mathcal{O}(h^2)right)end{equation}
      This is where I am unsure how to proceed. From this question, the top answer seems to suggest that it is a trivial matter to assume that we may assert $$f(t_n + 0.5h,y(t_n + 0.5h) + mathcal{O}(h^2)) = f(t_n + 0.5h,y(t_n + 0.5h)) + mathcal{O}(h^2) = y^prime(t_n+0.5h) + mathcal{O}(h^2)$$
      but this does not seem obvious to me. While it is stipulated that $f$ is an analytic function, I don't see how a perturbation in the input would necessarily perturb the output on the order of $h^2$ in general.



      If this is a good assumption to make, could someone please explain? If not, why? What is the correct way to proceed here?







      differential-equations numerical-methods approximation-theory






      share|cite|improve this question















      share|cite|improve this question













      share|cite|improve this question




      share|cite|improve this question








      edited Dec 31 '18 at 22:52







      Kyle Poe

















      asked Dec 31 '18 at 22:13









      Kyle PoeKyle Poe

      83




      83






















          1 Answer
          1






          active

          oldest

          votes


















          0














          In general you assume the Lipschitz condition, from there you get $|f(t,y+O(h^2))-f(t,y)|=O(h^2)$.



          For an order $p$ method to get the order $p$ local error you also need in general that $f$ is $p$ times continuously differentiable in all variables. Or at least $p-1$ times differentiable, with the highest derivative Lipschitz continuous.





          You should probably also use the central difference quotient for the first terms,
          $$
          y(t+h)-y(h)=hy'(t+0.5h)+O(h^3),
          $$

          then there are no second order terms in the formula that have to be compensated for.






          share|cite|improve this answer























          • Thank you. This makes sense, I wasn't thinking about an argument based on magnitude of the Lipschitz constant.
            – Kyle Poe
            Dec 31 '18 at 23:00











          Your Answer





          StackExchange.ifUsing("editor", function () {
          return StackExchange.using("mathjaxEditing", function () {
          StackExchange.MarkdownEditor.creationCallbacks.add(function (editor, postfix) {
          StackExchange.mathjaxEditing.prepareWmdForMathJax(editor, postfix, [["$", "$"], ["\\(","\\)"]]);
          });
          });
          }, "mathjax-editing");

          StackExchange.ready(function() {
          var channelOptions = {
          tags: "".split(" "),
          id: "69"
          };
          initTagRenderer("".split(" "), "".split(" "), channelOptions);

          StackExchange.using("externalEditor", function() {
          // Have to fire editor after snippets, if snippets enabled
          if (StackExchange.settings.snippets.snippetsEnabled) {
          StackExchange.using("snippets", function() {
          createEditor();
          });
          }
          else {
          createEditor();
          }
          });

          function createEditor() {
          StackExchange.prepareEditor({
          heartbeatType: 'answer',
          autoActivateHeartbeat: false,
          convertImagesToLinks: true,
          noModals: true,
          showLowRepImageUploadWarning: true,
          reputationToPostImages: 10,
          bindNavPrevention: true,
          postfix: "",
          imageUploader: {
          brandingHtml: "Powered by u003ca class="icon-imgur-white" href="https://imgur.com/"u003eu003c/au003e",
          contentPolicyHtml: "User contributions licensed under u003ca href="https://creativecommons.org/licenses/by-sa/3.0/"u003ecc by-sa 3.0 with attribution requiredu003c/au003e u003ca href="https://stackoverflow.com/legal/content-policy"u003e(content policy)u003c/au003e",
          allowUrls: true
          },
          noCode: true, onDemand: true,
          discardSelector: ".discard-answer"
          ,immediatelyShowMarkdownHelp:true
          });


          }
          });














          draft saved

          draft discarded


















          StackExchange.ready(
          function () {
          StackExchange.openid.initPostLogin('.new-post-login', 'https%3a%2f%2fmath.stackexchange.com%2fquestions%2f3058079%2fclarification-of-assumptions-made-in-deriving-error-of-implicit-midpoint-rule%23new-answer', 'question_page');
          }
          );

          Post as a guest















          Required, but never shown

























          1 Answer
          1






          active

          oldest

          votes








          1 Answer
          1






          active

          oldest

          votes









          active

          oldest

          votes






          active

          oldest

          votes









          0














          In general you assume the Lipschitz condition, from there you get $|f(t,y+O(h^2))-f(t,y)|=O(h^2)$.



          For an order $p$ method to get the order $p$ local error you also need in general that $f$ is $p$ times continuously differentiable in all variables. Or at least $p-1$ times differentiable, with the highest derivative Lipschitz continuous.





          You should probably also use the central difference quotient for the first terms,
          $$
          y(t+h)-y(h)=hy'(t+0.5h)+O(h^3),
          $$

          then there are no second order terms in the formula that have to be compensated for.






          share|cite|improve this answer























          • Thank you. This makes sense, I wasn't thinking about an argument based on magnitude of the Lipschitz constant.
            – Kyle Poe
            Dec 31 '18 at 23:00
















          0














          In general you assume the Lipschitz condition, from there you get $|f(t,y+O(h^2))-f(t,y)|=O(h^2)$.



          For an order $p$ method to get the order $p$ local error you also need in general that $f$ is $p$ times continuously differentiable in all variables. Or at least $p-1$ times differentiable, with the highest derivative Lipschitz continuous.





          You should probably also use the central difference quotient for the first terms,
          $$
          y(t+h)-y(h)=hy'(t+0.5h)+O(h^3),
          $$

          then there are no second order terms in the formula that have to be compensated for.






          share|cite|improve this answer























          • Thank you. This makes sense, I wasn't thinking about an argument based on magnitude of the Lipschitz constant.
            – Kyle Poe
            Dec 31 '18 at 23:00














          0












          0








          0






          In general you assume the Lipschitz condition, from there you get $|f(t,y+O(h^2))-f(t,y)|=O(h^2)$.



          For an order $p$ method to get the order $p$ local error you also need in general that $f$ is $p$ times continuously differentiable in all variables. Or at least $p-1$ times differentiable, with the highest derivative Lipschitz continuous.





          You should probably also use the central difference quotient for the first terms,
          $$
          y(t+h)-y(h)=hy'(t+0.5h)+O(h^3),
          $$

          then there are no second order terms in the formula that have to be compensated for.






          share|cite|improve this answer














          In general you assume the Lipschitz condition, from there you get $|f(t,y+O(h^2))-f(t,y)|=O(h^2)$.



          For an order $p$ method to get the order $p$ local error you also need in general that $f$ is $p$ times continuously differentiable in all variables. Or at least $p-1$ times differentiable, with the highest derivative Lipschitz continuous.





          You should probably also use the central difference quotient for the first terms,
          $$
          y(t+h)-y(h)=hy'(t+0.5h)+O(h^3),
          $$

          then there are no second order terms in the formula that have to be compensated for.







          share|cite|improve this answer














          share|cite|improve this answer



          share|cite|improve this answer








          edited Dec 31 '18 at 23:20

























          answered Dec 31 '18 at 22:54









          LutzLLutzL

          56.6k42054




          56.6k42054












          • Thank you. This makes sense, I wasn't thinking about an argument based on magnitude of the Lipschitz constant.
            – Kyle Poe
            Dec 31 '18 at 23:00


















          • Thank you. This makes sense, I wasn't thinking about an argument based on magnitude of the Lipschitz constant.
            – Kyle Poe
            Dec 31 '18 at 23:00
















          Thank you. This makes sense, I wasn't thinking about an argument based on magnitude of the Lipschitz constant.
          – Kyle Poe
          Dec 31 '18 at 23:00




          Thank you. This makes sense, I wasn't thinking about an argument based on magnitude of the Lipschitz constant.
          – Kyle Poe
          Dec 31 '18 at 23:00


















          draft saved

          draft discarded




















































          Thanks for contributing an answer to Mathematics Stack Exchange!


          • Please be sure to answer the question. Provide details and share your research!

          But avoid



          • Asking for help, clarification, or responding to other answers.

          • Making statements based on opinion; back them up with references or personal experience.


          Use MathJax to format equations. MathJax reference.


          To learn more, see our tips on writing great answers.




          draft saved


          draft discarded














          StackExchange.ready(
          function () {
          StackExchange.openid.initPostLogin('.new-post-login', 'https%3a%2f%2fmath.stackexchange.com%2fquestions%2f3058079%2fclarification-of-assumptions-made-in-deriving-error-of-implicit-midpoint-rule%23new-answer', 'question_page');
          }
          );

          Post as a guest















          Required, but never shown





















































          Required, but never shown














          Required, but never shown












          Required, but never shown







          Required, but never shown

































          Required, but never shown














          Required, but never shown












          Required, but never shown







          Required, but never shown







          Popular posts from this blog

          Can a sorcerer learn a 5th-level spell early by creating spell slots using the Font of Magic feature?

          Does disintegrating a polymorphed enemy still kill it after the 2018 errata?

          A Topological Invariant for $pi_3(U(n))$